Матричное представление углового момента

Мы должны дать матричное представление л С для электрона с л "=" 1 и с "=" 1 2 .

Я читаю л С как л С . Это верно? Тогда у нас было бы, например, для

л С ( | 1 , 1 | 1 / 2 , 1 / 2 ) "=" л | 1 , 1 С | 1 / 2 , 1 / 2 "=" 2 | 1 , 1 3 4 | 1 / 2 , 1 / 2 "=" 3 2 2 | 1 , 1 | 1 / 2 , 1 / 2 .

Эта поправка правильная? В этом случае я должен действовать таким же образом со всеми другими базисными векторами и записывать собственные значения по диагонали в матрице?

л и С являются векторами. л С является скаляром. Что л С ? Если вы хотите использовать тензорное произведение операторов, правильным выражением будет л С "=" к "=" 1 3 л я С я .
так вы говорите, что это бред? хорошо, вы знаете, как получить матричное представление для л С ?
Да: заменить каждого оператора л я и С я для соответствующей матрицы. Используйте тот факт, что л "=" 1 и с "=" 1 / 2 найти эти матрицы в учебниках.
Я этого не понимаю. Если мы скажем, что Дж "=" л + С , не значит ли это, что Дж "=" л я г + я г С ? Я думаю, что не понял всей концепции сложения угловых моментов. Какая правильная математическая запись для Дж "=" л + С ?
Правильное математическое обозначение этой суммы: Дж "=" л 1 вращаться + 1 сфера С , где 1 s — тождественные операторы в спиновом и орбитальном гильбертовом пространствах.
Дж "=" л + С на самом деле означает: Дж к "=" л к я с п я н + я о р б С к для к "=" 1 , 2 , 3 .
Спасибо. Итак, мне разрешено определять этот тензорный продукт только для каждого компонента? Но как это поможет мне оценить это л С Я имею в виду, я знаю только, что л 3 С 3 делает на канонической основе в квантовой механике, конечно л 1 С 1 и то же с индексом 2 полностью не определяются принципом неопределенности?
Поэтому л С "=" к ( л к я ) ( я С к ) "=" к л к С к .
Вы должны только явно записать матричную форму л С , насколько я понял. Эта матрица существует независимо от того, нет ли общих собственных векторов для всех компонент спина или углового момента.
да, но как найти эту матрицу? что мне нужно сделать сейчас?
Что касается спиновых матриц, т.к. с "=" 1 / 2 , они есть / 2 о я ХОРОШО?
Касательно л я , так как вы знаете, что л "=" 1 , это точно такие же матрицы, определяющие спиновые компоненты частицы спина 1 . Они написаны почти во всех учебниках.
en.wikipedia.org/wiki/Pauli_matrices , раздел Квантовая механика первые три матрицы (там л "=" Дж и л я "=" Дж я .
Очень тесно связано: physics.stackexchange.com/questions/60409/…

Ответы (1)

Есть две проблемы, которые необходимо решить, чтобы решить подобные проблемы.

  • Оба оператора углового момента являются векторными операторами , поэтому в некотором смысле они «принимают значения» в р 3 ; вас просят об их точечном продукте, который должен быть взят в этой копии р 3 . У вас была бы та же проблема, если бы вас попросили вычислить скалярный продукт. р п для одной частицы без спина.

  • Операторы орбитального и спинового углового момента действуют на два разных фактора тензорного произведения пространств Гильбета. Таким образом, любое (операторное) произведение скалярного орбитального оператора на скалярный спиновый оператор следует интерпретировать как тензорное произведение. У вас была бы та же проблема, если бы вас попросили рассчитать произведение л 2 С 2 , который необходимо интерпретировать как л 2 С 2 .

Таким образом, в вашем случае вы должны прочитать л С как

л С "=" я "=" 1 3 л я С я "=" я "=" 1 3 л я С я .
Чтобы вычислить матричное представление этого, вы должны начать с матричного представления каждого л я и С я . Затем вы вычисляете матрицы тензорных произведений л я С я . Наконец, вы складываете все эти матрицы вместе, чтобы получить окончательный результат.

Все это гораздо понятнее на примере. г компонента, например, легко, так как каждая матрица задается

л г "=" ( 1 0 0 0 0 0 0 0 1 ) и С г "=" 2 ( 1 0 0 1 ) ,
в базах { | 1 , | 0 , | 1 } и { | 1 2 , | 1 2 } соответственно. Тогда матрица тензорного произведения в базисе { | 1 | 1 2 , | 0 | 1 2 , | 1 | 1 2 , | 1 | 1 2 , | 0 | 1 2 , | 1 | 1 2 } , дан кем-то
л г С г "=" 2 2 ( 1 ( 1 0 0 0 0 0 0 0 1 ) 0 ( 1 0 0 0 0 0 0 0 1 ) 0 ( 1 0 0 0 0 0 0 0 1 ) 1 ( 1 0 0 0 0 0 0 0 1 ) ) "=" 2 2 ( 1 0 0 0 0 0 0 0 0 0 0 0 0 0 1 0 0 0 0 0 0 1 0 0 0 0 0 0 0 0 0 0 0 0 0 1 ) .
Эту процедуру следует повторить с обоими Икс и у компоненты. Каждый из них даст матрицу шесть на шесть (в данном случае). Чтобы получить окончательный ответ, вы должны сложить все три матрицы.